0 Daumen
567 Aufrufe

Aufgabe:

Wie kann ich diese Funktion in eine Geometrische/Potenz Reihe umwandeln?

\(\displaystyle x^{2} \ln (1+x) d x \)


Problem/Ansatz:

Habe überhaupt kein Plan. Ich muss diese Funktion irgendwie in diese Form bekommen:

\(\displaystyle \frac{1}{1-x}=\sum \limits_{k=0}^{\infty} x^{k} \)

Avatar von

Bitte das dx nicht beachten. Das steht da, weil ich erst die Funktion in eine geometrische Reihe umwandeln muss und dann das Integral der Reihe berechnen soll.

Wie kann ich diese Funktion...

Das ist keine Funktion.

Wenn es nicht die Form <linke Seite> = <rechte Seite> hat, ist es keine Funktion. Eine Funktion ist eine Zuordnungsvorschrift.

2 Antworten

+1 Daumen
 
Beste Antwort

Aloha :)$$f(x)=x^2\ln(1+x)=x^2\int\limits_{0}^x\frac{1}{1+y}\,dy=x^2\int\limits_0^x\frac{1}{1-(-y)}\,dy$$Für \(|y|<1\) ist der Integrand die Summe der unendlichen geometrischen Reihe. In diesem Konvergenzradius können wir Integration und Summation vertauschen:$$f(x)=x^2\int\limits_0^x\sum\limits_{n=0}^\infty (-y)^n\,dy=x^2\sum\limits_{n=0}^\infty(-1)^n\int\limits_0^x y^n\,dy=x^2\sum\limits_{n=0}^\infty(-1)^n\left[\frac{y^{n+1}}{n+1}\right]_{y=0}^x$$$$\phantom{f(x)}=x^2\sum\limits_{n=0}^\infty(-1)^n\frac{x^{n+1}}{n+1}$$Da \(x\) die obere Grenze der Integration über \(dy\) ist ist, muss \(|x|<1\) gelten, um die Forderung \(|y|<1\) einzuhalten. Damit sind wir fertig:$$f(x)=\sum\limits_{n=0}^\infty(-1)^n\frac{x^{n+3}}{n+1}\quad;\quad|x|<1$$

Avatar von 148 k 🚀

Vielen Dank Tschakamumba, aber meine Lösung ist im Nenner etwas anders?

blob.png

Text erkannt:

\( C+\sum \limits_{n=1}^{\infty}(-1)^{n} \frac{x^{n+3}}{n(n+3)}, R=1 \)

Ich habe meine Lösung bei Wolfram verglichen:

blob.png

Bei deiner Lösung gibt es gar keinen \(x^3\)-Term... Ohne Rechenweg kann ich nicht sagen, was das falsch gelaufen ist.

Achso, das ist eigentlich die Lösung aus meinem Buch.

Dann ist die Lösung aus deinem Buch falsch.

Ich bin aber noch etwas verwirrt.

blob.png

Text erkannt:

\( \int x^{2} \ln (1+x) d x \)

Muss man nicht am Ende Ihre Lösung noch einmal integrieren, damit die Reihe dem oben stehenden entspricht?

Ja, wenn ein Integralzeichen davor steht, muss auf jeden Fall nochmal integriert werden. Aber in der ursprünglichen Frage fehlt das Integralzeichen.

blob.png

Text erkannt:

\( \int x^{2} \ln (1+x) d x \)

Wäre dann:

blob.png

Text erkannt:

\( \frac{(-1)^{n} x^{n+4}}{(n+1)(n+4)}+C \)

Oder?

Hier musst du vorsichtig sein, da scheint mir noch eine Indexverschiebung drin zu sein. Die Summe in meiner Rechnung beginnt bei \(n=0\). Die Summe in der Buchlösung beginnt bei \(n=1\).

Ich integriere mal das Ergebnis aus meiner Antwort und wir schauen, was nach der Indexverschiebung rauskommt. Die Integrationskonstante \(C\) spare ich mir:$$F(x)=\int f(x)\,dx=\int\sum\limits_{n=0}^\infty(-1)^n\,\frac{x^{n+3}}{n+1}=\sum\limits_{n=0}^\infty(-1)^n\int\frac{x^{n+3}}{n+1}\,dx$$$$\phantom{F(x)}=\sum\limits_{n=0}^\infty(-1)^n\frac{x^{n+4}}{(n+1)(n+4)}$$Diese Lösung hast du angegeben.\(\quad\checkmark\)

Nun lassen wir die Summe bei \(n=1\) beginnen, dafür müssen wir alle \(n\) in den Summanden um \(1\) vermindren:$$F(x)=\sum\limits_{n=0\pink{+1}}^\infty(-1)^{n\pink{-1}}\,\frac{x^{n\pink{-1}+4}}{(n\pink{-1}+1)(n\pink{-1}+4)}=\sum\limits_{n=1}^\infty(-1)^{n-1}\,\frac{x^{n+3}}{n(n+3)}$$

Das ist aber immer noch nicht ganz die Lösung aus deinem Buch, denn wegen \((-1)^{n-1}=-(-1)^n\) müsste das Vorzeichen der gesamten Summe dann negativ sein.

+1 Daumen

Probier doch zunächst mal ln(x + 1) über ein Taylorpolynom n.-Grades darzustellen.

Hier noch ein Vergleichsergebnis für deine Aufgabe.

blob.png

Avatar von 479 k 🚀

Ein anderes Problem?

Stell deine Frage

Willkommen bei der Mathelounge! Stell deine Frage einfach und kostenlos

x
Made by a lovely community